Recherche sur le blog!

Affichage des articles dont le libellé est Centrale-Supélec. Afficher tous les articles
Affichage des articles dont le libellé est Centrale-Supélec. Afficher tous les articles

Concours Physique Centrale-Supélec (M, P') 1993 (Énoncé)

Centrale–Supélec, M, P’, 1993 (Physique I)
Énoncé
Ce problème comporte trois parties dont certaines questions peuvent être abordées de façon indépendante. La première partie abordera la propagation d’une onde de courant dans une ligne électrique, la deuxième précisera la structure du champ électromagnétique dans la ligne et la troisième traitera de la transmission d’une onde électromagnétique par une lame conductrice. Les données numériques sont regroupées en fin d’énoncé; on posera \(j^2 = -1\).

Onde de courant dans une ligne électrique

Une ligne électrique sans pertes est caractérisée par son coefficient d’inductance propre linéique et sa capacité linéique, respectivement notées \(L\) et \(C\). À l’abscisse \(x\) et à l’instant \(t\), on désigne par \(i(x,t)\) l’intensité du courant dans la ligne et par \(u(x,t)\) la tension entre les deux conducteurs de la ligne (cf. fig. [fig1]).
  1. Établir les deux équations différentielles liant \(i(x,t)\) et \(u(x,t)\).
  2. [I2] On cherche une solution de ces équations représentant une onde de courant de la forme \(i(x,t) = I(x) \exp \left(j \omega t\right)\) en notation complexe. Déterminer, dans ce cas, la forme la plus générale de \(i(x,t)\) et \(u(x,t)\). Exprimer en fonction des caractéristiques de la ligne la vitesse de phase \(v_\varphi\) de cette onde.
  3. La ligne, située dans l’espace \(x < 0\), s’étend jusqu’en \(x = 0\) où elle est fermée sur l’impédance \(Z_0\) (cf. fig. [fig2]). Montrer qu’il existe une valeur \(Z_c\) de \(Z_0\), appelée impédance caractéristique de la ligne telle que le rapport \(u/i\) devienne indépendant de \(x\). On exprimera \(Z_c\) en fonction de \(L\) et \(C\) et on précisera la forme de l’onde dans la ligne. Exprimer dans ce cas la puissance moyenne transportée par l’onde à l’abscisse \(x\). Que se passe-t-il physiquement en \(x = 0\)?
  4. La ligne s’étend maintenant jusqu’à \(x = + \infty\) mais on branche encore l’impédance \(Z_0 = Z_c\) en parallèle sur la ligne à l’abscisse \(x = 0\) (cf. fig. [fig3]). On s’intéresse à l’onde de courant dans la partie \(x < 0\) de la ligne.
    1. Montrer que cette onde voit en \(x = 0\) une impédance équivalente \(Z_1\) qui s’exprime très simplement en fonction de \(Z_c\).
    2. Définir et calculer le module \(r\) du coefficient de réflexion (en courant ou en tension) de l’onde en \(x = 0\).
  5. On place enfin sur la ligne précédente un court-circuit en parallèle à l’abscisse \(x = \ell\) (cf. fig. [fig4]).
    1. Quelle est la forme nécessaire de l’onde de courant entre les abscisses \(x=0\) et \(x=\ell\)?
    2. Montrer qu’il existe une valeur minimale \(\ell_0\) de \(\ell\) telle que le courant dans la partie positive de la ligne s’annule en \(x=0\). On exprimera \(\ell_0\) en fonction de la longueur d’onde \(\lambda\) de l’onde de courant dans la ligne. En déduire alors le coefficient de réflexion et la forme de l’onde dans la partie négative de la ligne.

Champ électromagnétique dans la ligne

La ligne précédente est constituée de deux rubans conducteurs parfaits, de faible épaisseur, de largeur \(a\), distants de \(b\), l’espace entre les rubans étant vide (cf. fig. [fig5]). Les rubans sont parcourus par des courants de densités surfaciques \(\vec j_s = j_s(x,t) \vec e_x\) et \(- \vec j_s\) et présentent entre leurs faces des densités surfaciques de charge \(\sigma(x,t)\) et \(- \sigma(x,t)\).
On étudie les champs \(\vec E\) et \(\vec B\) uniquement dans l’espace situé entre les rubans et on suppose que ces champs ne dépendent que l’abscisse \(x\) du point considéré et de l’instant \(t\). On néglige donc tout effet de bord.
  1. Exprimer, en fonction des constantes électromagnétiques du vide \(\varepsilon_0\) et \(\mu_0\) et des densités \(j_s\) et \(\sigma\) les champs \(\vec E(x,t)\) et \(\vec B(x,t)\) dans l’espace vide entre les rubans.
    On considère à nouveau dans toute la suite de cette partie [PartieII] une onde de courant dans la ligne, d’intensité de la forme \(i(x,t) = I \exp \left[j \left(\omega t - k x\right)\right]\) en notation complexe, où \(k\) est une constante positive et \(I\) une constante réelle.
  2. [II2] À partir des équations de Maxwell, exprimer deux relations liant \(\sigma(x,t)\) et \(i(x,t)\). En déduire la vitesse de phase \(v_\varphi\) de l’onde et montrer que la structure du champ électromagnétique est celle d’une onde plane dans le vide illimité.
  3. Déterminer l’énergie magnétique \({\mathrm{d}}\epsilon_B\) d’une tranche d’épaisseur \({\mathrm{d}}x\) de la ligne. En déduire le coefficient d’inductance propre \(L\) de la ligne.
  4. Déterminer l’énergie \({\mathrm{d}}\epsilon_E\) associée au champ électrique \(\vec E\) de la même tranche d’épaisseur \({\mathrm{d}}x\). En déduire la capacité linéique \(C\) de la ligne.
  5. Déduire des résultats précédents l’accord entre les questions [I2] et [II2] du problème quant à la vitesse de phase \(v_\varphi\).
  6. Exprimer le champ \(\vec E\) en fonction des dimensions de la ligne et de la tension \(u(x,t)\) entre les rubans. Peut-on écrire une relation de la forme \(\vec E = - {\overrightarrow{\mathrm{grad}}\,}V\) dans l’espace vide entre les rubans?
    On désire fermer la ligne sur son impédance \(Z_c\) en introduisant, entre les rubans, à l’abscisse \(x = 0\), une plaque conductrice de résistivité \(\varrho\), d’épaisseur \(e\), de largeur \(a\) et de longueur \(b\) (cf. fig. [fig6]).
  7. On considérera dans cette question que l’épaisseur \(e\) est suffisamment faible pour que l’on puisse admettre que le courant traversant la plaque soit réparti de manière uniforme.
    1. Déterminer \(Z_c\) en fonction de \(\varrho\), \(e\), \(a\) et \(b\). Montrer que la résistance \(R_c\) d’un carré de la plaque, de côté quelconque, s’exprime en fonction des seules constantes \(\varepsilon_0\) et \(\mu_0\). On appellera impédance adaptée au vide cette grandeur \(R_c\) dont on donnera la valeur numérique.
    2. On veut réaliser cette plaque avec:
      • du cuivre de résistivité \(\varrho = {1,7\cdot 10^{-8}}{\,\Omega\cdot\mathrm{m}}\);
      • du carbone de résistivité \(\varrho = {3,5\cdot 10^{-3}}{\,\Omega\cdot\mathrm{m}}\).
      Quel devrait être, dans chaque cas, l’épaisseur \(e\) de la plaque? Commenter.
  8. Déterminer le vecteur de Poynting associé à l’onde électromagnétique entre les rubans. Quelle est la puissance moyenne transportée par l’onde? Que se passe-t-il quand l’onde arrive en \(x = 0\), la ligne étant fermée par la plaque d’impédance \(Z_c\)?

Réflexion sur une plaque conductrice

On considère à présent une onde électromagnétique plane dans le vide illimité, de pulsation \(\omega\) qui a des caractéristiques identiques à celles étudiées dans la partie [PartieII]. On écrira les champs de cette onde:
\[\vec E_i = E_0 \exp \left[j \omega\left(t - \frac{x}{c}\right)\right] \vec e_y \hspace{2em} \vec B_i = \frac{E_0}{c} \exp \left[j \omega\left(t - \frac{x}{c}\right)\right] \vec e_z\]
\(c\) est la vitesse de la lumière dans le vide. À l’abscisse \(x = 0\) (cf. fig. [fig7]) on place une plaque conductrice plane infinie, orthogonale à \(\vec e_x\), de constantes électromagnétiques égales à celles du vide \(\varepsilon_0\) et \(\mu_0\), d’épaisseur \(e\) et de résistivité \(\varrho\) identiques à celles calculées dans la partie précédente: un carré de côté quelconque de la plaque a donc une résistance \(R_c\) adaptée au vide.
  1. Expliquer qualitativement pourquoi il existera pourtant une onde réfléchie sur la plaque. En vous inspirant des résultats précédents et en argumentant votre réponse, pouvez-vous indique sans calculs quel sera le module \(r\) du coefficient de réflexion de cette onde sur la plaque?
    On se propose de retrouver ce résultat directement à partir de l’étude des ondes dans le vide et la plaque. Pour ce faire, on rappelle que, moyennant l’approximation \(\varrho\varepsilon_0\omega \ll 1\) supposée ici vérifiée, le champ électrique dans la plaque conductrice est de la forme:
    \[\vec E_\varrho = \left\{A_1 \exp \left(- \frac{x}{\delta}\right) \exp \left[j \left(\omega t - \frac{x}{\delta}\right)\right] + A_2 \exp \left(\frac{x}{\delta}\right) \exp \left[j \left(\omega t + \frac{x}{\delta}\right)\right] \right\} \vec e_y\]
    \(A_1\) et \(A_2\) sont des constantes déterminées par les conditions aux limites de la plaque et \(\delta\) une distance caractéristique du conducteur et de l’onde, appelée profondeur de peau, et qui vaut \(\displaystyle \delta = \sqrt{\frac{2\varrho}{\mu_0\omega}}\).
  2. Expliquer d’où provient l’approximation indiquée et préciser le champ magnétique \(\vec B_\varrho\) associé dans la plaque. Justifier l’expression de \(\delta\).
    \(\left(\vec E_i, \vec B_i\right)\) étant l’onde incidente arrivant sur la plaque et \(\left(\vec E_\varrho, \vec B_\varrho\right)\) l’onde se propageant dans la plaque, on désigne par \(\left(\vec E_r, \vec B_r\right)\) l’onde réfléchie sur la plaque et \(\left(\vec E_t, \vec B_t\right)\) l’onde transmise dans l’espace \(x > e\).
    On écrira \(\vec E_r\) et \(\vec E_t\) sous la forme:
    \[\vec E_r = \alpha E_0 \exp \left[j \omega\left(t + \frac{x}{c}\right)\right] \vec e_y \hspace{2em} \vec E_t = \tau E_0 \exp \left[j \omega\left(t - \frac{x}{c}\right)\right] \vec e_y\]
  3. Déterminer quatre relations liant \(\alpha\), \(\tau\), \(A_1\) et \(A_2\).
  4. Montrer que l’approximation précédente implique également qu’on ait \(e \ll \delta\). En déduire, après simplifications des relations, la valeur de \(\alpha\).
  5. Que faudrait-il placer, et à quel endroit, pour annuler l’onde réfléchie? On pourra d’abord répondre qualitativement en s’appuyant sur des résultats précédents et démontrer ensuite le résultat recherché.
Formulaire et données numériques:
Formule d’analyse vectorielle \({\overrightarrow{\mathrm{rot}}\,}{\overrightarrow{\mathrm{rot}}\,}\vec u = {\overrightarrow{\mathrm{grad}}\,}{\mathrm{div}\,}\vec u - \Delta \vec u\)
Célérité de la lumière dans le vide \(c = {3,00\cdot 10^{8}}{\,\mathrm{m}\cdot\mathrm{s}^{-1}}\)
Perméabilité magnétique du vide \(\mu_0 = {4\cdot\pi\cdot 10^{-7}}{\,\mathrm{H}\cdot\mathrm{m}^{-1}}\)

Concours Physique Centrale-Supélec (M, P') 1991 Physique II (Corrigé)

Corrigé centrale 91 M-P'
Première partie.
I- Collision neutron-noyau
1/ Conservation de la qdm : $m{\vec V_1} = m{\vec V_2} + M{\vec w_2} \Rightarrow {\vec V_1} = {\vec V_2} + A{\vec w_2}$
Conservation de l'énergie: $\text{ }\!\!{\scriptscriptstyle 1\!/\!{ }_2}\!\!\text{ }m\vec{V}_{1}^{2}=\text{ }\!\!{\scriptscriptstyle 1\!/\!{ }_2}\!\!\text{ }m\vec{V}_{2}^{2}+\text{ }\!\!{\scriptscriptstyle 1\!/\!{ }_2}\!\!\text{ }M\vec{w}_{2}^{2}\Rightarrow \vec{V}_{1}^{2}=\vec{V}_{2}^{2}+A\vec{w}_{2}^{2}$
2/ De ${\vec V_1} = {\vec V_2} + A{\vec w_2}$, on tire : $\vec V_2^2 = {({\vec V_1} - A{\vec w_2})^2} = \vec V_1^2 + A\vec w_2^2 - 2A{V_1}{w_2}\cos \theta $
Soit $\cos \theta = \frac{{\vec V_1^2 - \vec V_2^2 + {A^2}\vec w_2^2}}{{2A{V_1}{w_2}}} = \frac{{A\vec w_2^2 + {A^2}\vec w_2^2}}{{2A{V_1}{w_2}}} = \frac{{{w_2}}}{{{V_1}}}\frac{{1 + A}}{2}$> 0 donc 0 < θ < π/2
En fonction des énergies : $\text{ }\!\!{\scriptscriptstyle 1\!/\!{ }_2}\!\!\text{ }m\vec{V}_{1}^{2}=\text{ }\!\!{\scriptscriptstyle 1\!/\!{ }_2}\!\!\text{ }m\vec{V}_{2}^{2}+\text{ }\!\!{\scriptscriptstyle 1\!/\!{ }_2}\!\!\text{ }M\vec{w}_{2}^{2}\Rightarrow {{E}_{1}}-{{E}_{2}}=\text{ }\!\!{\scriptscriptstyle 1\!/\!{ }_2}\!\!\text{ A}\,\text{m}\,\vec{w}_{2}^{2}$ et ${{E}_{1}}=\text{ }\!\!{\scriptscriptstyle 1\!/\!{ }_2}\!\!\text{ }\,\text{m}\,\vec{V}_{1}^{2}$
Alors $\cos \theta = \frac{{{w_2}}}{{{V_1}}}\frac{{1 + A}}{2} = \sqrt {\frac{{{E_1} - {E_2}}}{{A{E_1}}}} \frac{{1 + A}}{2}$donc $\frac{{{E_2}}}{{{E_1}}} = 1 - \frac{{4A{{\cos }^2}\theta }}{{{{(1 + A)}^2}}}$

II- Modèle des sphères dures.
1/ La force de contact passe par le centre d'inertie, donc la vitesse ${\vec w_2}$ sera dirigé suivant la réaction normale. On en déduit : $\sin \theta = \frac{b}{{{R_1} + {R_2}}}$
2/ Le paramètre d'impact peut varier entre 0 et la valeur R1 + R2. Ce qui correspond pour le centre du neutron à
à une cible de surface variant de 0 à (R1 + R2)2.
La probablité de recevoir un impact sur une couronne de rayon : b → b + db est :$\frac{{dP}}{1} = \frac{{2\pi bdb}}{{\pi {{({R_1} + {R_2})}^2}}}$
3/ Par définition: $ < - Ln\,[1 - K{\cos ^2}\theta ]{ > _b} = < - Ln\,[1 - \frac{{K{b^2}}}{{{{({R_1} + {R_2})}^2}}}]{ > _b} = - \int\limits_0^{{R_1} + {R_2}} {Ln[1 - \frac{{K{b^2}}}{{{{({R_1} + {R_2})}^2}}}]\;db} $
En posant $x = \frac{{K{b^2}}}{{{{({R_1} + {R_2})}^2}}}$⇒ $\frac{1}{K}\left[ {(1 - x)Ln(1 - x) - (1 - x)} \right]_0^K = \frac{1}{K}\left[ {(1 - K)Ln(1 - K) - (1 - K) + 1} \right]$
Ce qui donne : $1 + \frac{{1 - K}}{K}Ln(1 - K)$ cqfd . Il faut que 0 < K < 1 pour que la fonction aît un sens.
4/ On a obtenu $\frac{{{E_2}}}{{{E_1}}} = 1 - \frac{{4A{{\cos }^2}\theta }}{{{{(1 + A)}^2}}} = 1 - K{\cos ^2}\theta $ avec $K = \frac{{4A}}{{{{(1 + A)}^2}}}$< 1 si A > 1
on peut utiliser le résultat précédent : $K = \frac{{4A}}{{{{(1 + A)}^2}}} \Rightarrow 1 - K = {\left( {\frac{{A - 1}}{{A + 1}}} \right)^2}$
Donc coefficient de ralentissement : $\gamma = < - Ln\,[\frac{{{E_2}}}{{{E_1}}}]{ > _b} = 1 + {\left( {\frac{{1 - A}}{{\sqrt {2A} }}} \right)^2}Ln(\frac{{A - 1}}{{A + 1}}) = $
5/ a)La dérivée de γ vaut zéro pour : $0 = \left( {\frac{{1 - A}}{{\sqrt A }}} \right)\left\{ { - \left( {\frac{{{A^{1/2}} + {A^{ - 1/2}}}}{{2\sqrt 2 \;A}}} \right)Ln(\frac{{A - 1}}{{A + 1}}) - \left( {\frac{1}{{\sqrt A }}} \right)\left( {\frac{1}{{(A + 1)}}} \right)} \right\}$
Le terme entre crochet ne s'annulant pas, la racine est A = 1. On vérifiera que c'est bien un maximum pour le ralentissement.
b) A-N : 1H (A = 1) γ = 1 ; 2H (A = 2) γ = 0,725 ; 12C (A = 12) γ = 0,158 ; 238U (A = 238) γ = 0,008 ;
III- Application aux ralentissements des neutrons.
1/ Il y a ½ kT par degré de liberté, donc E300K = 3/2kT = 3,9.10−2 eV.
C'est très faible devant l'énergie initiale des neutrons. On peut considèrer les noyaux immobiles, sauf pour les dernières collisions.
2 a/ Avec $\gamma = < - Ln\,[\frac{{{E_2}}}{{{E_1}}}]{ > _b}$ et en écrivant : $\frac{{{E_n}}}{{{E_0}}} = \frac{{{E_n}}}{{{E_{n - 1}}}}\frac{{{E_{n - 1}}}}{{{E_{n - 2}}}}\; \cdots \frac{{{E_1}}}{{{E_0}}} \Rightarrow Ln\left( {\frac{{{E_n}}}{{{E_0}}}} \right) = \sum\limits_1^n {Ln\left( {\frac{{{E_p}}}{{{E_{p - 1}}}}} \right)} $
on a en raisonnant sur les valeurs moyennes : $Ln\left( {\frac{{{E_n}}}{{{E_0}}}} \right) = - n\gamma \Rightarrow {E_n} = {E_0}{e^{ - \gamma }}$
2b/ $n = - \frac{1}{\gamma }Ln\left( {\frac{{{E_{300K}}}}{{{E_0}}}} \right)$d'où 1H : n = 17 ; 2H : n = 24 ; 12C : n = 108 ; 238U : n = 214;
3a/ A une date t : $v(t) = \sqrt {\frac{{2E(t)}}{m}} $, la durée moyenne intercollision est: $\Delta t = \frac{\lambda }{{v(t)}}$et le nombre de collisions par unité de temps est : $\frac{{dn}}{{dt}} = \frac{1}{{\Delta t}} \Rightarrow \frac{{dn}}{{dt}} = \frac{1}{\lambda }\sqrt {\frac{{2E}}{m}} $.
3b/ L'équation $Ln\left( {\frac{{{E_n}}}{{{E_0}}}} \right) = - n\gamma $donne, en passant à la limite : $\gamma \,dn = - Ln\,[\frac{{E + dE}}{E}] = - \frac{{dE}}{E}$
soit : $\gamma \frac{{dt}}{\lambda }\sqrt {\frac{{2E}}{m}} = - \frac{{dE}}{E}$ ; en posant $\,y = \frac{E}{{\;{E_0}}}$ on a $\gamma \frac{{dt}}{\lambda }\sqrt {\frac{{2{E_0}}}{m}} = - \frac{{dy}}{{\;{y^{3/2}}}}$
3c/ L'intégration conduit à : $2\left[ {{y^{ - 1/2}} - 1} \right] = \frac{\gamma }{\lambda }t\,\sqrt {\frac{{2{E_0}}}{m}} $soit : $\,\sqrt {\frac{{{E_0}}}{E}} = 1 + \frac{\gamma }{{2\lambda }}t\,\sqrt {\frac{{2{E_0}}}{m}} $
4a/ On calcule d'abord $\,\sqrt {\frac{{{E_0}}}{E}} \approx 5000$ puis avec γ = 0,158 on trouve t = 120 µs .
On a toujours : $\,\sqrt {\frac{{{E_0}}}{E}} > > 1$ donc $\,t = \frac{{2\lambda }}{\gamma }\sqrt {\frac{m}{{2E}}} $ indépendant de E0.
4b/ La distance parcourue pendant dt est : $dx = v.dt = dt\sqrt {\frac{{2E}}{m}} $ et on a aussi $\gamma \frac{{dt}}{\lambda }\sqrt {\frac{{2E}}{m}} = - \frac{{dE}}{E}$
donc $dx = - \frac{\lambda }{\gamma }\frac{{dE}}{E} \Rightarrow x = \frac{\lambda }{\gamma }Ln\,{\frac{{{E_0}}}{E}_{300K}}$ on trouve ainsi x = 2,8 m.
On peut remarquer que cette distance corespond à nλ puisque $n = - \frac{1}{\gamma }Ln\left( {\frac{{{E_{300K}}}}{{{E_0}}}} \right)$.

Deuxième partie.
1a/ Avec $\xi \,\vec u = {A_1}M \to $ ⇒ le théorème d'Ampère donne$\vec B = \frac{{{\mu _0}I}}{{2\pi {\xi ^2}}}\vec k \wedge \xi \vec u$
1b/${A_1}M \to = $$(r - a\cos \theta ){\vec u_r} + a\sin \theta {\vec u_{^\theta }}$⇒$\vec B = \frac{{{B_0}}}{{{\xi ^2}}}\left\{ \begin{array}{l} - a\sin \theta \;{{\vec u}_r}\\(r - a\cos \theta \;){{\vec u}_\theta }\end{array} \right.$et${\xi ^2} = {a^2} + {r^2} - 2\,a\,r\cos \theta $
1c/ $\vec B' = {B_0}\left\{ \begin{array}{l} - \frac{{a\sin \theta }}{{{\xi ^2}}}\; = - \left[ {\sin \theta {\rm{ + 2}}u\sin \theta \;\cos \theta - {{\rm{u}}^{\rm{2}}}\sin \theta [1 - 4{{\cos }^2}\theta {\rm{]}}} \right]{\rm{ }}\\\frac{{(r - a\cos \theta \;)}}{{{\xi ^2}}} = \left[ {u - {\rm{cos}}\theta - 2{\rm{u}}\,{\rm{co}}{{\rm{s}}^{\rm{2}}}\theta + {{\rm{u}}^{\rm{2}}}\cos \theta [3 - 4{{\cos }^2}\theta ]} \right]\end{array} \right.$
2a/ Il faut faire une rotation de π et changer le signe du courant. Soit: $\vec{B}''(u,\theta )=-\vec{B}'(u,\theta +\pi )$
2b/ ${B_{1r}} = B{'_r}(u,\theta ) - B{'_r}(u,\theta + \pi ) = - 2{B_0}\left[ {\sin \theta - {{\rm{u}}^{\rm{2}}}\sin \theta [1 - 4{{\cos }^2}\theta {\rm{]}}} \right]$
${B_{1\theta }} = B{'_\theta }(u,\theta ) - B{'_\theta }(u,\theta + \pi ) = - 2{B_0}\left[ {{\rm{cos}}\theta - {{\rm{u}}^{\rm{2}}}\cos \theta [3 - 4{{\cos }^2}\theta ]} \right]$
en linéarisant : ${B_{1r}} = - 2{B_0}\left[ {\sin \theta + {{\rm{u}}^{\rm{2}}}\sin 3\theta } \right]$ et${B_{1\theta }} = - 2{B_0}\left[ {{\rm{cos}}\theta + {{\rm{u}}^{\rm{2}}}\cos 3\theta } \right]$
3a/ Il faut faire une rotation d'angle − 2π/3 et d'angle +2π/3 .
3b/ Donc ${B_r} = {B_{1r}}(u,\theta ) + {B_{1r}}(u,\theta - 2\pi /3) + {B_{1r}}(u,\theta + 2\pi /3)$
${B_\theta } = {B_{1\theta }}(u,\theta ) + {B_{1\theta }}(u,\theta - 2\pi /3) + {B_{1\theta }}(u,\theta + 2\pi /3)$
Or $\left\{ \begin{array}{l}\cos (\theta - 2\pi /3) + \cos (\theta + 2\pi /3) = - \cos \theta \\\sin (\theta - 2\pi /3) + \sin (\theta + 2\pi /3) = - \sin \theta \end{array} \right.$on a finalement:
${B_r} = - 2{B_0}\left[ {3{{\rm{u}}^{\rm{2}}}\sin 3\theta } \right]$
${B_\theta } = - 2{B_0}\left[ {3{{\rm{u}}^{\rm{2}}}\cos 3\theta } \right]$ donc $C = 6$
4a/ Ligne de champ: $d\vec \ell //\vec B \Rightarrow \frac{{dr}}{{rd\theta }} = \frac{{{B_r}}}{{{B_\theta }}}$ ⇒$\frac{{dr}}{r} = \frac{{\sin 3\theta }}{{\cos 3\theta }}d\theta \Rightarrow \,{r^3} = r_0^3/\cos 3\theta $
4b/ ci-contre : allure des lignes de champ.
4c/ Module $B(r) = 6{B_0}\;{r^2}/{a^2}$,
lignes isomodules B(r) = Cte sur un cercle de centre O
II- Action du champ sur un neutron
1a/ Pour un dipôle donc deux cas possibles : ${{E}_{//}}=-\,B$ et ${{E}_{\bot }}=\,B$
Soit en remplaçant B par $C{B_0}\;{r^2}/{a^2}$⇒ ${{E}_{//}}=-\text{ }\!\!{\scriptscriptstyle 1\!/\!{ }_2}\!\!\text{ }m{{\Omega }^{2}}{{r}^{2}}$ et ${{E}_{}}=\text{ }\!\!{\scriptscriptstyle 1\!/\!{ }_2}\!\!\text{ }m{{\Omega }^{2}}{{r}^{2}}$
1b/ La force est donnée par : $\vec F = \, - gr\vec ad\,{E_p}$ donc ${\vec F_{//}} = m{\Omega ^2}\,\vec r$ et ${\vec F_{\rlap{--} \rlap{--} \not /\rlap{--} /}} = - m{\Omega ^2}\,\vec r$
Pour confiner il faut une force de rappel, seuls les neutrons antiparallèles peuvent être confinés.
2a/ La RFD donne : ${\vec F_{\rlap{--} \rlap{--} \not /\rlap{--} /}} = - m{\Omega ^2}\,\vec r = m\frac{{{d^2}\vec r}}{{d{t^2}}} + m\frac{{{d^2}z}}{{d{t^2}}}\vec k$ ⇒$ - m{\Omega ^2}\,\vec r = m\frac{{{d^2}\vec r}}{{d{t^2}}}{\rm{ et }}\frac{{{d^2}z}}{{d{t^2}}} = 0$
2b/ L'intégration donne :$\,\vec r(t) = {\vec A_1}\cos \,\Omega t + {\vec A_2}\sin \Omega t$ où ${\vec A_1}{\rm{ et }}{\vec A_2}$ sont des constantes.
soit avec les conditions initiales: $z = {v_0}t$ et $\,\vec r(t) = {x_0}\vec i\cos \,\Omega t + \frac{{{u_0}}}{\Omega }\vec j\sin \Omega t$.
2c/ La trajectoire est une hélice d'axe Oz et de section elliptique.
3a/ Le neutron est confiné si le grand axe de l'ellipse est inférieur au rayon a; x0 étant plus petit que a il faut que:$a > \frac{{{u_0}}}{\Omega }$ soit encore :${u_C} = a\,\Omega $.
3b/ A-N: uC = 5,9 m.s−1ce qui donne EC = 18.10−8 eV et aussi TC = 1,4.10−3 K
Ce résultat justifie l'appellation neutron ultra-froids.
3c/ La fonction de répartition de Boltzmann permet de calculer la fraction de neutrons qui ont une énergie inférieure à la valeur calculée précédemment:
$F = \int\limits_0^{{E_C}} {\frac{1}{{\sqrt {2\pi } }}\frac{1}{{{{(kT)}^{3/2}}}}\sqrt E \exp ( - E/kT)\,dE} $
si T = 300 K << TC on peut simplifier ⇒$F \approx \int\limits_0^{{E_C}} {\frac{1}{{\sqrt {2\pi } }}\frac{1}{{{{(kT)}^{3/2}}}}\sqrt E \,dE} = \frac{1}{{\sqrt {2\pi } }}\frac{1}{{{{(kT)}^{3/2}}}}\frac{2}{3}\left[ {{E^{3/2}}} \right]_0^{{E_C}}$
Soit finalement : $F = \sqrt {\frac{3}{{4\pi }}} {\left[ {\frac{{{T_C}}}{T}} \right]^{3/2}} \approx {5.10^{ - 9}}$ donc extrémement faible.
4/ Les neutrons ont un mouvement de dérive suivant l'axe Oz. or les fils créant le champ magnétique ne peuvent être rééllement infinis. Le confinement n'a lieu que dans la partie centrale du dispositif et se termine lorsque les neutrons sortent du dispositif.
III- Amélioration du confinement

1a/ Pour les neutrons confinés : ${\vec F_{//}} = - m{\Omega ^2}\,\vec r$ avec maintenant $\vec r = $$O'M \to $$ = (\rho - R){\vec u_\rho } + z\vec k$
1b/ En cylindriques : $\vec a = (\ddot \rho - \rho {\dot \theta ^2}){\vec u_\rho } + (2\dot \rho \dot \theta + \rho \ddot \theta ){\vec u_\theta } + \ddot z\vec k$
1c/ Equations du mouvement : $\left\{ \begin{array}{l}\ddot \rho - \rho {{\dot \theta }^2} = - {\Omega ^2}(\rho - R)\\2\dot \rho \dot \theta + \rho \ddot \theta = 0\\\ddot z = - {\Omega ^2}z\end{array} \right.$
2a/ Compte tenu des conditions initiales: $\ddot z = - {\Omega ^2}z \Rightarrow z = {z_0}\cos (\Omega t) + \frac{{{V_0}}}{\Omega }\sin \Omega t$.
2b/ $2\dot \rho \dot \theta + \rho \ddot \theta = \frac{1}{\rho }\frac{{d({\rho ^2}\dot \theta )}}{{dt}} = 0 \Rightarrow {\rho ^2}\dot \theta = Cte = \rho _0^2{\omega _0}$ "mouvement projeté sur x0y à force centrale".
2c/ Il reste l'équation en ρ(t): $\ddot \rho - \rho {\dot \theta ^2} = \ddot \rho - \left( {\frac{{\rho _0^4\omega _0^2}}{{{\rho ^3}}}} \right) = - {\Omega ^2}(\rho - R)$
3a/ si ω0 = 0 alors θ = θ0 est constant : $\ddot \rho = - {\Omega ^2}(\rho - R) \Rightarrow (\rho - R) = ({\rho _0} - R)\cos \Omega t$,
c'est l'équation paramètrique (z(t),ρ(t)) d'une ellipse de centre O'.
3b/ si $\dot \theta = Cte = {\omega _0}$ alors ${\rho ^2} = \rho _0^2$, la trajectoire est sinusoïde dessinée sur un cylindre d'axe Oz.
La trajectoire sera fermée si la durée d'un tour est un multiple de la période, soit $\Omega = n{\omega _0}$.
4a/ Si $\rho = {\rho _m}[1 + \varepsilon (t)]$ alors l'équation en ε est :${\rho _m}\ddot \varepsilon - \left( {\frac{{\rho _0^4\omega _0^2}}{{\rho _m^3}}} \right)[1 - 3\varepsilon ] = - {\Omega ^2}({\rho _m} - R + {\rho _m}\varepsilon )$
4b/ La valeur moyenne correspond à ε = 0 : $ - \left( {\frac{{\rho _0^4\omega _0^2}}{{\rho _m^3}}} \right) = - {\Omega ^2}({\rho _m} - R)$on a
4c/ Par différence : ${\rho _m}\ddot \varepsilon + 3\left( {\frac{{\rho _0^4\omega _0^2}}{{\rho _m^3}}} \right)\varepsilon + {\Omega ^2}{\rho _m}\varepsilon = 0$ soit : $\ddot \varepsilon + 3\left( {\frac{{\rho _0^4\omega _0^2}}{{\rho _m^4}}} \right)\varepsilon + {\Omega ^2}\varepsilon = 0$
ce qui s'intègre en $\varepsilon (t) = {\varepsilon _0}\cos (\Omega 't + {\varphi _0})$ en posant : $\Omega ' = \sqrt {3\left( {\frac{{\rho _0^4\omega _0^2}}{{\rho _m^4}}} \right) + {\Omega ^2}} $.
Ce qui donne alors la vitesse angulaire: $\dot \theta = \frac{{\rho _0^2{\omega _0}}}{{{\rho ^2}}} \approx \frac{{\rho _0^2{\omega _0}}}{{\rho _m^2}}[1 - 2\varepsilon ]$.
4d/ Les trajectoires sont alors ses oscillations autour des sinusoïdes tracées sur un cylindre. La vitesse angulaire étant elle même oscillante.
5/ La pesanteur entaîne un mouvement de chute selon l'équation z = ½ gt2 qui s'ajoute aux oscillations. Au bout d'une période la "chute" vaut donc : $h = 2g{\pi ^2}/{\Omega ^2}$ .
On calcule alors : h = 5,6 mm, ce qui n'est pas négligeable.
___________________________

Concours Physique Centrale-Supélec (M) 1988 (Corrigé)

Corrigé de physique I M du concours de Centrale 1988

I.a)

$m\frac{{d\vec v}}{{dt}} = q\vec v \wedge \vec B$.

I.b)

Notons $\omega  = \frac{{qB}}{m} = \varepsilon {\omega _c}$ ; nous utiliserons la notation $\omega $ dans la suite à la place de la notation $\varepsilon {\omega _c}$ de l’énoncé.
$\begin{array}{l}\left( 1 \right) & {{\dot v}_x} = \omega {v_y}\\\left( 2 \right) & {{\dot v}_y} =  - \omega {v_x}\\\left( 3 \right) & {{\dot v}_z} = 0\end{array}$

I.c)

D’après l’équation (3), ${v_z} = {v_{//0}}$ est constant au cours du temps.
Posons $u = {v_x} + i{v_y}$ ; en formant la combinaison $\left( 1 \right) + i\left( 2 \right)$, $\dot u =  - i\omega u$, d’où, compte tenu de $u\left( 0 \right) = {v_{ \bot 0}}$ , $u = {v_{ \bot 0}}\exp \left( { - i\omega t} \right)$.

I.d)

Soit $r = x + i\omega  = \int {udt = \frac{{i{v_{ \bot 0}}}}{\omega }} \exp \left( { - i\omega t} \right) + cste$.
La particule a un mouvement hélicoïdal uniforme qui résulte de la composition de deux mouvements : un mouvement circulaire de rayon ${\rho _L} = \frac{{\left| {{v_{ \bot 0}}} \right|}}{{{\omega _c}}}$ avec la vitesse angulaire $ - \omega $ dans un plan perpendiculaire au champ magnétique, le centre $G$ de ce cercle décrivant un mouvement

 rectiligne uniforme de vitesse ${\vec v_{//}}$ parallèle au champ magnétique.

I.e)

Pour un électron :
$\begin{array}{l}{\omega _c} = \frac{{eB}}{m} = \frac{{1,6 \times {{10}^{ - 19}} \times 5}}{{9,1 \times {{10}^{ - 31}}}} = 8,79 \times {10^{11}}rad.{s^{ - 1}}\\{v_ \bot } = \sqrt {\frac{{2E}}{m}}  = \sqrt {\frac{{2 \times 1,6 \times {{10}^{ - 15}}}}{{9,1 \times {{10}^{ - 31}}}}}  = 5,93 \times {10^7}m.{s^{ - 1}}\\{\rho _L} = \frac{{{v_ \bot }}}{{{\omega _c}}} = 6,75 \times {10^{ - 5}}m\end{array}$
Pour un proton :
$\begin{array}{l}{\omega _c} = \frac{{eB}}{{{m_H}}} = \frac{{1,6 \times {{10}^{ - 19}} \times 5}}{{1,67 \times {{10}^{ - 27}}}} = 4,79 \times {10^8}rad.{s^{ - 1}}\\{v_ \bot } = \sqrt {\frac{{2E}}{{{m_H}}}}  = \sqrt {\frac{{2 \times 1,6 \times {{10}^{ - 15}}}}{{1,67 \times {{10}^{ - 27}}}}}  = 1,38 \times {10^6}m.{s^{ - 1}}\\{\rho _L} = \frac{{{v_ \bot }}}{{{\omega _c}}} = 2,89 \times {10^{ - 3}}m\end{array}$


II.a)

$m\frac{{d\vec v}}{{dt}} = q\vec v \wedge \vec B + q\vec E$.
 $\begin{array}{l}\left( 4 \right) & {{\dot v}_x} = \omega {v_y} + \frac{{q{E_x}}}{m}\\\left( 5 \right) & {{\dot v}_y} =  - \omega {v_x}\\\left( 6 \right) & {{\dot v}_z} = \frac{{q{E_z}}}{m}\end{array}$
D’après l’équation (6), ${v_z} = \frac{{q{E_z}}}{m}t + {v_{//0}}\quad ;\quad z = \frac{{q{E_z}}}{{2m}}{t^2} + {v_{//0}}t + cste$.
Posons $u = {v_x} + i{v_y}$ ; en formant la combinaison $\left( 4 \right) + i\left( 5 \right)$, on obtient $\dot u + i\omega u = \frac{{q{E_x}}}{m}$, d’où, compte tenu de $u\left( 0 \right) = {v_{ \bot 0}}$ , $u = \left( {{v_{ \bot 0}} + \frac{{i{E_x}}}{B}} \right)\exp \left( { - i\omega t} \right) - \frac{{i{E_x}}}{B}$.
Soit $r = x + iy = \int {udt = \left( {\frac{{i{v_{ \bot 0}}}}{\omega } - \frac{{m{E_x}}}{{q{B^2}}}} \right)} \exp \left( { - i\omega t} \right) - \frac{{i{E_x}}}{B}t + cste$.

II.b)


 La particule décrit un cercle de rayon ${\rho _L} = \left| {\frac{{i{v_{ \bot 0}}}}{\omega } - \frac{{m{E_x}}}{{q{B^2}}}} \right| = \frac{1}{{{\omega _c}}}\sqrt {v_{ \bot 0}^2 + \frac{{E_x^2}}{{{B^2}}}} $ avec la vitesse angulaire $ - \omega $ (comme l’indique la dérivée $ - i\omega $ de l’argument de l’exponentielle complexe), le centre $G$ de ce cercle décrivant un mouvement uniformément varié de vitesse $ - \frac{{{E_x}}}{B}{\vec u_y} + \left( {{v_{//0}} + \frac{{q{E_z}}}{m}t} \right){\vec u_z}$.

II.c)

${\vec v_{ \bot G}} =  - \frac{{{E_x}}}{B}{\vec u_y} = \frac{{\vec E \wedge \vec B}}{{{B^2}}}$.

III.a)

${\vec v_{ \bot G}} = \frac{{\vec F \wedge \vec B}}{{q{B^2}}}$.

III.b)

${\vec v_{ \bot G}} = \frac{{m\vec g \wedge \vec B}}{{q{B^2}}}$

III.c)

Il y a création d’un courant de densité $\vec j = \sum {nq\vec v}  = \sum {n\frac{{m\vec g \wedge \vec B}}{{{B^2}}}}  = n\left( {m + M} \right)\frac{{\vec g \wedge \vec B}}{{{B^2}}}$ ; en pratique, ce courant est négligeable, parce que $n$ est petit.


IV.a)

$\vec B\left( M \right) = \vec B\left( G \right) + \left( {y - {y_G}} \right)\frac{{dB}}{{dy}}\left( G \right){\vec u_z}$
$\vec F = q\vec v \wedge \vec B = q\vec v \wedge B\left( G \right){\vec u_z} + q\vec v \wedge \left( {y - {y_G}} \right)\frac{{dB}}{{dy}}\left( G \right){\vec u_z} = q\vec v \wedge B\left( G \right){\vec u_z} + q\frac{{dB}}{{dy}}\left( G \right)\left( {y - {y_G}} \right)\left( {\dot y{{\vec u}_x} - \dot x{{\vec u}_y}} \right)$

IV.b)

L’équation différentielle du mouvement étant non linéaire, on la résout approximativement. En première approximation, $\vec F = q\vec v \wedge \vec B$, d’où $x = {x_G} + {\rho _L}\cos \omega t$, $y = {y_G} - {\rho _L}\sin \omega t$, $G$ ayant un mouvement rectiligne uniforme parallèle à $\vec B$.
Dans une meilleure approximation, on considère une force supplémentaire. Compte tenu de $\left\langle {\vec v} \right\rangle  = \vec 0$, le terme principal est $\left\langle {\vec F} \right\rangle  = q\frac{{dB}}{{dy}}\left( G \right)\left\langle {\left( {y - {y_G}} \right)\left( {\dot y{{\vec u}_x} - \dot x{{\vec u}_y}} \right)} \right\rangle $.
$\begin{array}{l}\dot x \approx  - {\rho _L}\omega \sin \omega t\quad \dot y =  - {\rho _L}\omega \cos \omega t\\\left\langle {\left( {y - {y_G}} \right)\dot y} \right\rangle  = \rho _L^2\omega \left\langle {\cos \omega t\sin \omega t} \right\rangle  = 0\\\left\langle {\left( {y - {y_G}} \right)\dot x} \right\rangle  = \rho _L^2\omega \left\langle {{{\sin }^2}\omega t} \right\rangle  = \frac{1}{2}\rho _L^2\omega \\\left\langle {\vec F} \right\rangle  =  - \frac{1}{2}\rho _L^2\omega q\frac{{dB}}{{dy}}\left( G \right){{\vec u}_y} =  - \frac{{mv_L^2}}{{2B}}\vec \nabla B\end{array}$
Cette expression montre que la force est dirigée dans la direction où le module du champ magnétique décroît le plus vite, quelle que soit la charge ou la vitesse.

IV.c)

Appliquons l’expression de la vitesse de dérive de III.a en y remplaçant la force par sa valeur moyenne :
${\vec v_{ \bot G}} =  - \frac{{mv_ \bot ^2\left( {\vec \nabla B \wedge \vec B} \right)}}{{2q{B^3}}}$
Cette expression, équivalente à celle proposée par l’énoncé, puisque ${\rho _L} = \left| {\frac{{m{v_L}}}{{qB}}} \right|$, lui est préférable, car elle a un signe bien défini.

V.

Tous les champs magnétiques de révolution n’ont pas nécessairement la forme proposée. Par exemple, le champ magnétique d’une nappe d’un courant régulièrement réparti sur un tore d’axe $Oz$ est de révolution autour de cet axe, mais est de la forme ${B_\theta }\left( {r,z} \right){\vec u_\theta }$. Il faut faire l’hypothèse supplémentaire que tout plan contenant $Oz$ est un plan de symétrie du champ magnétique ; alors $\vec B = {B_r}\left( {r,z} \right){\vec u_r} + {B_z}\left( {r,z} \right){\vec u_z}$.
Notons aussi que, contrairement à la formulation de l’énoncé, $\vec B$ n’est pas une fonction de $r$ et $z$seuls : il dépend aussi de $\theta $ par l’intermédiaire de ${\vec u_r}$.

V.a)

Une spire d’axe $Oz$crée un tel champ magnétique. En effet, tout plan contenant $Oz$ est un plan d’antisymétrie du courant donc un plan de symétrie du champ magnétique, donc ${B_\theta } = 0$. D’autre part, la distribution de courant est invariante par rotation autour de $Oz$, donc les coordonnées du champ magnétique ne dépendent pas de $\theta $ : $\vec B = {B_r}\left( {r,z} \right){\vec u_r} + {B_z}\left( {r,z} \right){\vec u_z}$.

V.b)

Supposons que le champ magnétique ne présente pas de singularité sur l’axe. Exprimons approximativement $\vec B$ au voisinage de l’axe par un développement en puissances successives de $r$ tronqué à l’ordre 1. Comme $Oz$ est un axe de révolution du champ magnétique, c’est un axe de symétrie : ${B_z}\left( {r,z} \right)$ est une fonction paire de $r$ et ${B_r}\left( {r,z} \right)$ est une fonction impaire de $r$ ; le développement tronqué à l’ordre 1 est de la forme ${B_z}\left( {r,z} \right) \approx {B_z}\left( {0,z} \right)$ et ${B_r}\left( {r,z} \right) \approx r\frac{{\partial {B_r}}}{{\partial r}}\left( {0,z} \right)$.
Soit une surface fermée formée d’un cylindre d’axe $Oz$, de rayon $r$ petit et de longueur $dz$ complété par deux disques terminaux de rayons $r$ et d’abscisses $z$ et $z + dz$. Le flux du champ magnétique à travers cette surface fermée est nul :
$\mathop{{\int\!\!\!\!\!\int}\mkern-21mu \bigcirc} {\vec B \cdot \overrightarrow {dS} }  = {B_z}\left( {z + dz} \right)\pi {r^2} - {B_z}\left( z \right)\pi {r^2} + 2\pi rdz{B_r}\left( r \right) = \frac{{d{B_z}\left( {0,z} \right)}}{{dz}}\pi {r^2}dz + \frac{{\partial {B_r}}}{{\partial r}}\left( {0,z} \right)2\pi {r^2}dz = 0$ ; d’où $\frac{{\partial {B_r}}}{{\partial r}}\left( {0,z} \right) =  - \frac{1}{2}\frac{{d{B_z}\left( {0,z} \right)}}{{dz}}$ et près de l’axe ${B_r} \approx  - \frac{r}{2}\frac{{d{B_z}\left( {0,z} \right)}}{{dz}}$.

V.c)

$m\frac{{d{v_z}}}{{dt}} = {\left( {q\vec v \wedge \vec B} \right)_z} =  - q{v_\theta }{B_r} = \frac{{q{v_\theta }r}}{2}\frac{{d{B_z}}}{{dz}} =  - \frac{{mv_ \bot ^2}}{{2{B_z}}}\frac{{d{B_z}}}{{dz}} \Rightarrow \frac{{d{v_{//}}}}{{dt}} =  - \frac{{v_ \bot ^2}}{{2{B_z}}}\frac{{d{B_z}}}{{dz}}$ (puisque $r =  - \frac{{m{v_\theta }}}{{qB}}$).

V.d)

La théorème de la puissance cinétique s’écrit :
 $\begin{array}{l}\frac{d}{{dt}}\left( {\frac{1}{2}m\left( {v_z^2 + v_ \bot ^2} \right)} \right) = q\left( {\vec v \wedge \vec B} \right) \cdot \vec v\\m{v_z}\frac{{d{v_z}}}{{dt}} + \frac{m}{2}\frac{{dv_ \bot ^2}}{{dt}} = 0\\ - {v_z}\frac{{v_ \bot ^2}}{{2{B_z}}}\frac{{d{B_z}}}{{dz}} + \frac{1}{2}\frac{{dv_ \bot ^2}}{{dt}} = 0\\ - \frac{{v_ \bot ^2}}{{{B_z}}}\frac{{d{B_z}}}{{dt}} + \frac{{dv_ \bot ^2}}{{dt}} = 0\\\frac{{dv_ \bot ^2}}{{v_ \bot ^2}} - \frac{{d{B_z}}}{{{B_z}}} = 0\\d\ln \left( {v_ \bot ^2/{B_z}} \right) = 0\\v_ \bot ^2/{B_z} = cste\\\mu  = \frac{{mv_ \bot ^2}}{{2{B_z}}} = cste\end{array}$
$\mu $ est le moment du dipôle magnétique équivalent à la particule chargée pour un ou plusieurs tours : $\vec \mu  = \frac{1}{2}\overrightarrow {GM}  \wedge q\vec v$.

V.e)

$\mu  = \frac{{m{r^2}{\omega ^2}}}{{2{B_z}}} = \frac{{{q^2}}}{{2m}}{r^2}{B_z} = cste$, donc le flux du champ magnétique $\pi {r^2}{B_z}$ à travers le cercle décrit par la particule autour de $G$ est constant : la trajectoire de la particule est une hélice qui s’enroule sur un tube de champ d’axe $Oz$.


VI.a)

Comme on a supposé ${\rho _L} <  < R$, une ligne de champ est presque rectiligne et on peut lui appliquer localement les résultats de V.e. On pourrait le faire sur une grande distance s’il existait une force égale à $m\frac{{v_{//}^2}}{R}{\vec u_n}$ , où ${\vec u_n}$ est le vecteur unitaire de la normale principale à la ligne de champ. En l’absence d’une telle force, le champ magnétique est la source d’une force $ - m\frac{{v_{//}^2}}{R}{\vec u_n}$ qui d’après II.a crée la vitesse de dérive ${\vec v'_{ \bot G}} =  - \frac{{mv_{//}^2{{\vec u}_n} \wedge \vec B}}{{Rq{B^2}}}$.

VI.b)

Cette proposition est-elle vraie en toute généralité ? Peut-être.
Supposons que les lignes de champ soient des cercles de même axe. La question posée est alors est un problème de géométrie plane. Soit une ligne de champ, $M$ un de ses points, $C$, $R$ et ${\vec u_n}$ le centre de courbure, le rayon de courbure et le vecteur unitaire de la normale principale en $M$. Appliquons le théorème d’Ampère à une courbe fermée $ADEFA$, où $AD$   est un arc de cette ligne de champ vu de $C$ sous l’angle $d\alpha $, $DE$ et $FA$ deux segments appartenant à des droites passant par $C$ et $EF$ un arc d’une ligne de champ voisine. D’après le théorème d’Ampère, $\oint\limits_{ADEFA} {\vec B \cdot d\vec r}  = 0$, soit $B\left( A \right).CA.d\alpha  - B\left( F \right).CF.d\alpha  = 0 \Rightarrow {\left( {\overrightarrow {grad} B} \right)_n} = \frac{{B\left( F \right) - B\left( A \right)}}{{AF}} = \frac{{B\left( A \right)\left( {\frac{{CA}}{{CF}} - 1} \right)}}{{AF}} = \frac{{B\left( A \right)}}{{CF}}$, d’où ${\left( {\overrightarrow {grad} B} \right)_n} = \frac{B}{R}$.

VI.c)

Pour effectuer le calcul, il faudrait connaître la composante de $\overrightarrow {grad} B$ sur la binormale à la ligne de champ. Supposons qu’elle soit nulle (c’est vrai dans le cas traité à la question précédente), ${\vec v''_{ \bot G}} =  - \frac{{mv_ \bot ^2}}{{2Rq{B^2}}}{\vec u_n} \wedge \vec B$, d’où ${\vec v_{ \bot G}} =  - \frac{{m\left( {v_{//}^2 + v_ \bot ^2/2} \right)}}{{Rq{B^2}}}{\vec u_n} \wedge \vec B$.
Remarque : comme $\frac{{qB}}{m} = \omega $, cette formule est homogène, car de la forme ${\vec v_{ \bot G}} =  - \frac{{\left( {v_{//}^2 + v_ \bot ^2/2} \right)}}{{R\omega }}{\vec u_n} \wedge \frac{{\vec B}}{B}$. Si $R >  > {\rho _L}$ (cas usuel), ${v_ \bot } >  > {v_{ \bot G}}$.

VII.a)

Il y a conservation de l’énergie cinétique $\frac{1}{2}m\left( {v_z^2 + v_ \bot ^2} \right) = \frac{1}{2}m\left( {v_{z0}^2 + v_{ \bot 0}^2} \right)$ et du moment dipolaire $\frac{{v_ \bot ^2}}{B} = \frac{{v_{ \bot 0}^2}}{{{B_0}}}$. Si le champ magnétique croît, $v_ \bot ^2$ croît, ${v_{//}}$ décroît et donc peut s’annuler ; si c’est le cas, il change de signe par la suite, car $v_{//}^2$ ne peut devenir négatif : la particule est réfléchie.

VII.b)

$\sin \theta  = {v_ \bot }/v$$v$ est constant et $\frac{{v_ \bot ^2}}{B} = \frac{{v_{ \bot 0}^2}}{{{B_0}}}$, d’où $\frac{{{{\sin }^2}\theta }}{B} = \frac{{{{\sin }^2}{\theta _0}}}{{{B_0}}}$.

VII.c) et d)

La particule est réfléchie quand $\sin \theta  = 1$.
Elle l’est au niveau de $S$ ou $S'$ si ${\theta _0} = {\theta _{0m}} = \arcsin \sqrt {\frac{{{B_0}}}{{{B_{0m}}}}} $.
Si ${\theta _0} < {\theta _{0m}}$, la particule n’est pas réfléchie : elle est dans le cône de perte.
Si ${\theta _0} > {\theta _{0m}}$, la particule est réfléchie : les deux bobinages se comportent comme des miroirs magnétiques.

VII.e)

La durée annoncée par l’énoncé paraît bien grande. C’est la durée moyenne entre collisions qui régit la durée de confinement, le temps pour aller d’un miroir à l’autre étant beaucoup plus petit 
Si les probabilités de l’orientation de la vitesse après une collision sont également réparties dans toutes les directions, la probabilité que la direction de la vitesse soit dans l’un des deux cônes de perte est $\frac{{2 \times 2\pi \left( {1 - \cos {\theta _{0m}}} \right)}}{{4\pi }} = 1 - \cos {\theta _{0m}}$ ; la durée de confinement est $\frac{{{t_c}}}{{1 - \cos {\theta _{0m}}}}$.

VIII.a)

Le théorème d’Ampère appliqué à un cercle d’axe $Oz$ et de rayon $\rho  = R + r\cos \theta $ donne ${B_\phi }\left( {r,\theta } \right) = \frac{{{\mu _0}NI}}{{2\pi \left( {R + r\cos \theta } \right)}} = \frac{{{B_0}}}{{1 + \left( {r/R} \right)\cos \theta }}$.

VIII.b)

${\vec v_{ \bot G}} =  - \frac{{m\left( {v_{//}^2 + v_ \bot ^2/2} \right)}}{{Rq{B^2}}}{\vec u_n} \wedge \vec B = \frac{{m\left( {v_{//}^2 + v_ \bot ^2/2} \right)}}{{RqB}}{\vec u_z}$.
Les ions sont éjectés dans la direction et le sens de $Oz$ et les électrons dans le sens contraire. Leur vitesse de dérive est la même en moyenne : ${v_{ \bot G}} = \frac{{mv_ \bot ^2}}{{eBR}} = \frac{{2 \times 1,6 \times {{10}^{ - 15}}}}{{1,6 \times {{10}^{ - 19}} \times 5}} = 4{\kern 1pt} 000m.{s^{ - 1}}$. La durée de confinement est de l’ordre de $\frac{{2{r_m}}}{{{v_{ \bot G}}}} = \frac{{2 \times 0,2}}{{4000}} = {10^{ - 4}}s$.


IX.a)

Déterminons le champ magnétique créé par un courant de densité $\vec j = {j_\phi }\left( r \right){\vec u_\phi }$.
Tout plan contenant $Oz$ est un plan d’antisymétrie du courant, donc un plan de symétrie du champ magnétique, donc ${B_\phi } = 0$. La distribution de courant est invariante dans les rotations d’axe $Oz$. D’où $\vec B = {B_r}\left( {r,\theta } \right){\vec u_r} + {B_\theta }\left( {r,\theta } \right){\vec u_\theta }$.
Or l’énoncé suppose ${B_r} = 0$ ($\vec B = {\vec B_\phi } + {\vec B_\theta }$ à la question IX.b), ce que la symétrie ne permet pas de conjecturer.
Il faut donc considérer que le champ magnétique est voisin de celui d’un courant cylindrique tangent au courant ${\vec j_\phi }\left( r \right){\vec u_\phi }$ pour la valeur de $\phi $ considérée. Cette approximation paraît acceptable si ${r_m} <  < R$, ce que nous supposerons.
Si $M$ est le point pour lequel $r = 0$ dans le plan de coordonnée azimutale $\phi $ considérée, et si $Mz'$ est la tangente au cercle d’axe $Oz$  passant par $M$, la nouvelle distribution de courant a la symétrie cylindrique par rapport à $Mz'$ : tout plan contenant $Mz'$ est plan de symétrie du courant, donc d’antisymétrie du champ magnétique, donc $\vec B$ est orthoradial ; cette distribution de courant est invariante par rotation autour de $Mz'$, donc $\vec B = {B_\theta }\left( r \right){\vec u_\theta }$. Enfin, ${B_\theta }\left( r \right)$ ne dépend pas de $\phi $, car la distribution exacte de courant est invariante par rotation autour de $Oz$. Appliquons le théorème d’Ampère à un cercle d’axe $Mz'$ et de rayon $r$ : $2\pi r{B_\theta } = {\mu _0}I\left( r \right)$ d’où ${B_\theta } = \frac{{{\mu _0}I\left( r \right)}}{{2\pi r}}$.

IX.b)

Un petit déplacement $\left( {dr,rd\theta ,\left( {R + r\cos \theta } \right)d\phi } \right)$ le long d’une ligne de champ est parallèle au champ magnétique $\left( {0,{B_\theta },{B_\phi }} \right)$ ; pour ce déplacement :
·         $dr = 0$ : toute ligne de champ fait partie d’un tore dont la section est un cercle concentrique avec la section du solénoïde toroïdal ;
·         $\frac{{\left( {R + r\cos \theta } \right)d\phi }}{{rd\theta }} = \frac{{{B_\phi }}}{{{B_\theta }}} = \frac{{\frac{{{B_0}}}{{1 + \left( {r/R} \right)\cos \theta }}}}{{\frac{{{\mu _0}I\left( r \right)}}{{2\pi r}}}} \Rightarrow \frac{{d\phi }}{{d\theta }} = \frac{{2\pi {r^2}{B_0}}}{{{\mu _0}RI\left( r \right){{\left( {1 + \left( {r/R} \right)\cos \theta } \right)}^2}}}$.
Cette équation est de la forme $\frac{{d\phi }}{{d\theta }} = q\left( r \right) = \frac{{2\pi {r^2}{B_0}}}{{{\mu _0}RI\left( r \right)}}$ si on néglige les termes d’ordre 1 et suivants en $r/R$, ce qui est conforme à l’approximation qui nous a permis de calculer le champ magnétique.
$q\left( r \right)$ est le rapport entre le nombre de tours que fait une ligne de champ dans la direction azimutale et le nombre tours qu’elle fait dans la direction poloïdale.
Remarque : dans les cas simples, les lignes de champ magnétiques sont des courbes fermées. Ici, ce n’est le cas que si  $q\left( r \right)$ est un entier, ce qui est peu probable, d’autant que les calculs sont approximatifs.
Si $q\left( 0 \right) = 1$, $I\left( r \right) \approx {j_\phi }\left( 0 \right)\pi {r^2}$, d’où ${j_\phi }\left( 0 \right) = \frac{{2{B_0}}}{{{\mu _0}R}} = \frac{{2 \times 5}}{{4\pi  \times {{10}^{ - 7}}}} = 8 \times {10^6}A.{m^{ - 2}}$.

IX.c)

En première approximation, une particule chargée tourne autour de son centre guide, qui se meut le long d’une ligne de champ ; toutefois, le centre guide dérive lentement perpendiculairement à cette ligne de champ, avec une vitesse telle que la force magnétique associée neutralise la force moyenne sur un tour $\left\langle {\vec F} \right\rangle  =  - \frac{{mv_{//}^2{{\vec u}_n}}}{{{R_{courbure}}}} - \frac{{mv_ \bot ^2\vec \nabla B}}{{2B}}$, où ${R_{courbure}}$ est le rayon de courbure d’une ligne de champ. Qualitativement, cette force est dirigée dans la direction opposée à celle de la projection du point considéré sur le cercle moyen du tore ; la dérive crée un mouvement qui s’enroule autour de ce cercle moyen du tore. Les particules ont deux raisons de décrire des hélices autour de ce cercle, cette dérive et le fait qu’elles suivent les lignes de champ.
L’énoncé demande de mettre en évidence « que l'effet de dérive est compensé exactement entre les portions de trajectoire du centre guide, situées de part et d'autre du plan équatorial du tore ». Voici en figure 1 la trajectoire sans champ poloïdal et en figure 2 la trajectoire avec champ poloïdal ; ces deux figures sont dilatées dans le sens de l’axe $z$, pour mieux montrer la dérive :

En l’absence de champ poloïdal, les particules s’évadent en partant dans une direction parallèle à $Oz$ ; le champ poloïdal crée un gradient de champ magnétique ; s’il est supérieur à celui produit par la courbure du tore, alors, après avoir tourné de 180° autour du cercle moyen du tore, les particules prennent une dérive opposée, aussi elles oscillent autour du cercle moyen du tore. L’énoncé suggère que les particules oscillent autour du plan équatorial du tore ; en fait, c’est vrai, mais ce n’est pas un bon argument pour comprendre la stabilité.
En raison du gradient du champ magnétique dû à la distance à l’axe $Oz$, en réalité les trajectoires sont centrées par rapport à un cercle un peu plus grand que le cercle moyen du tore.

IX.d)

Le module du champ magnétique $\sqrt {{{\left( {\frac{{{B_0}}}{{1 + \left( {r/R} \right)\cos \theta }}} \right)}^2} + {{\left( {\frac{{{\mu _0}I\left( r \right)}}{{2\pi r}}} \right)}^2}} $ varie sur la trajectoire parce que $\theta $ varie. D’après la question VII, les particules peuvent être piégées et osciller sur une ligne de champ entre deux positions où le champ magnétique est assez grand pour les réfléchir.

IX.e)

Pour ces particules, l’effet de dérive n’est pas compensé, car elles ne sont pas également dans toutes les directions autour du cercle moyen, aussi la dérive due au gradient du champ magnétique a une direction moyenne et ne se compense pas. Notons aussi que le sens de la dérive ne dépend pas du sens de la composante de la vitesse parallèle au champ magnétique et donc que cette dérive ne se compense pas sur un aller et sur le retour suivant.


Concours Physique Centrale-Supélec M, P' 1993 (Corrigé)

Centrale-Supelec 1993 - Physique M et P'

Partie I- Onde de courant dans une ligne électrique:

I.1) Equations différentielles liant i(x,t) et u(x,t):

Loi des noeuds: $i(x,t)-i(x+dx,t)=Cdx\frac{\partial u}{\partial t}\Rightarrow C\frac{\partial u}{\partial t}=-\frac{\partial i}{\partial x}$
Loi des mailles: $u(x,t)=u(x+dx,t)+Ldx\frac{\partial i}{\partial t}\Rightarrow \frac{\partial u}{\partial x}=-L\frac{\partial i}{\partial t}$

I.2) Vitesse de phase et expressions de i(x,t) et de u(x,t):

On en déduit les équations de propagation: $\frac{{{\partial }^{2}}u}{\partial {{x}^{2}}}-\frac{1}{LC}\frac{{{\partial }^{2}}u}{\partial {{t}^{2}}}=0\ \ et\ \ \frac{{{\partial }^{2}}i}{\partial {{x}^{2}}}-\frac{1}{LC}\frac{{{\partial }^{2}}i}{\partial {{t}^{2}}}=0$
La vitesse de phase est ${{v}_{\varphi }}=\frac{1}{\sqrt{LC}}$; la fonction I(x) vérifie l'équation $\frac{{{d}^{2}}I}{d{{x}^{2}}}+LC{{\omega }^{2}}I=0$ dont l'équation caractéristique ${{p}^{2}}+LC{{\omega }^{2}}=0$ admet les racines $p=\pm j\sqrt{LC}\omega =\pm j\frac{\omega }{{{v}_{\varphi }}}$; d'où:
$I(x)={{I}_{1}}{{e}^{-j\omega \frac{x}{{{v}_{\varphi }}}}}+{{I}_{2}}{{e}^{j\omega \frac{x}{{{v}_{\varphi }}}}}$ et $i(x,t)={{I}_{1}}{{e}^{j\omega (t-\frac{x}{{{v}_{\varphi }}})}}+{{I}_{2}}{{e}^{j\omega (t+\frac{x}{{{v}_{\varphi }}})}}$
Une des deux équations différentielles liant u et i permet d'obtenir l'expression de u(x,t):
$u(x,t)=\frac{1}{{{v}_{\varphi }}c}\left( {{I}_{1}}{{e}^{j\omega (t-\frac{x}{{{v}_{\varphi }}})}}-{{I}_{2}}{{e}^{j\omega (t+\frac{x}{{{v}_{\varphi }}})}} \right)$

I.3) Impédance caractéristique:

En x=0: $u={{Z}_{0}}i$; or à l'abscisse x<0 l'impédance est définie par $Z(x)=\frac{u(x,t)}{i(x,t)}=\frac{1}{{{v}_{\varphi }}c}\frac{{{I}_{1}}{{e}^{-j\omega \frac{x}{{{v}_{\varphi }}}}}-{{I}_{2}}{{e}^{j\omega \frac{x}{{{v}_{\varphi }}}}}}{{{I}_{1}}{{e}^{-j\omega \frac{x}{{{v}_{\varphi }}}}}+{{I}_{2}}{{e}^{j\omega \frac{x}{{{v}_{\varphi }}}}}}$; Z(x) est indépendante de x si =0 ${{Z}_{c}}=\frac{1}{{{v}_{\varphi }}c}=\sqrt{\frac{L}{C}}$
et $Z(0)={{Z}_{c}}$. Alors l'onde est progressive et se propage en sens >0 de x'x à la célérité ${{v}_{\varphi }}$.
$p=\operatorname{Re}(u)\operatorname{Re}(i)=\frac{1}{4}(u+{{u}^{*}})(i+{{i}^{*}})$; mais < ui>=0 et $<{{u}^{*}}{{i}^{*}}>$=0; avec $u={{Z}_{c}}i$, on obtient: $P=<p>=\frac{1}{2}{{Z}_{c}}{{\left| u \right|}^{2}}=\frac{I_{1}^{2}}{2{{Z}_{c}}}$. En x=0, la puissance moyenne est dissipée dans l'impédance caractéristique ${{Z}_{0}}$, réelle, par effet Joule.

I.4.a) Impédance équivalente:

${{Z}_{1}}={{Z}_{c}}//{{Z}_{c}}\Rightarrow {{Z}_{1}}=\frac{{{Z}_{c}}}{2}$

I.4.b)Coefficient de reflexion en intensité:

Par définition: $r=\frac{{{i}_{r}}(0,t)}{{{i}_{i}}(0,t)}$(coefficient de réflexion) et $t=\frac{{{i}_{t}}(0,t)}{{{i}_{i}}(0,t)}$(coefficient de transmission)
Continuité de la tension: ${{u}_{i}}+{{u}_{r}}={{u}_{t}}\Rightarrow {{Z}_{c}}{{i}_{i}}-{{Z}_{c}}{{i}_{r}}={{Z}_{c}}{{i}_{t}}\Rightarrow {{i}_{i}}-{{i}_{r}}={{i}_{t}}$ et $1-r=t$
Loi des noeuds: ${{i}_{i}}+{{i}_{r}}=2{{i}_{t}}({{i}_{t}}\ dans\ {{Z}_{c}}$ et dans la ligne à droite)$\Rightarrow 1+r=2t$; on en déduit $r=\frac{1}{3}$.

I.5.a) Forme de l'onde de courant entre x=0 et x=l:

On a la superposition de deux ondes progressives de sens de propagation opposés:
$i={{I}_{t}}{{e}^{j\omega (t-\frac{x}{{{v}_{\varphi }}})}}+{{I}_{r}}{{e}^{j\omega (t+\frac{x}{{{v}_{\varphi }}})}}$ et $\frac{u}{{{Z}_{c}}}={{I}_{t}}{{e}^{j\omega (t-\frac{x}{{{v}_{\varphi }}})}}-{{I}_{r}}{{e}^{j\omega (t+\frac{x}{{{v}_{\varphi }}})}}$
En x=l, u=0 (court-circuit): ${{I}_{t}}{{e}^{-j\omega \frac{l}{{{v}_{\varphi }}}}}={{I}_{r}}{{e}^{j\omega \frac{l}{{{v}_{\varphi }}}}}$$\Rightarrow i=2{{I}_{t}}{{e}^{j\omega (t-\frac{l}{{{v}_{\varphi }}})}}\cos \frac{\omega (l-x)}{{{v}_{\varphi }}}$

I.5.b) Coefficient de réflexion:

i=0 $\forall t\Rightarrow x=l-(2q+1)\frac{\lambda }{4},q\in Z$; on veut que ceci soit vrai en x=0 d'où $l=(2q+1)\frac{\lambda }{4}$; la valeur minimale possible est ${{l}_{0}}=\frac{\lambda }{4}$ et alors ${{I}_{r}}=-{{I}_{t}}$, u(0,t)=$2{{Z}_{c}}{{I}_{t}}{{e}^{j\omega t}}$; en écrivant la continuité de la tension et la loi des noeuds, on a: ${{u}_{i}}+{{u}_{r}}=2{{Z}_{c}}{{I}_{t}}{{e}^{j\omega t}}$ $\Rightarrow {{i}_{i}}-{{i}_{r}}=2{{I}_{t}}{{e}^{j\omega t}}$ et ${{i}_{i}}+{{i}_{r}}=2{{I}_{t}}{{e}^{j\omega t}}$ r=0.

Partie II-Champ électromagnétique dans la ligne:

II.1) Expression des champs en fonction des densités:

Le champ magnétique est selon z'z et le champ électrique selon y'y.Equation de passage du champ magnétique: ${{\vec{B}}_{2}}-{{\vec{B}}_{1}}={{\mu }_{0}}{{\vec{j}}_{s}}\wedge {{\vec{N}}_{12}}\Rightarrow B(x,t)={{\mu }_{0}}{{j}_{s}}(x,t)$ et $i(x,t)=a{{j}_{s}}\Rightarrow B(x,t)=\frac{{{\mu }_{0}}}{a}i(x,t)$;
Equation de passage du champ électrique: ${{\vec{E}}_{2}}-{{\vec{E}}_{1}}=\frac{\sigma (x,t)}{{{\varepsilon }_{0}}}{{\vec{N}}_{12}}\Rightarrow E(x,t)=\frac{\sigma }{{{\varepsilon }_{0}}}$;

II.2) Equations différentielles vérifiées par les densités:

L'équation de Maxwell-Faraday donne: $\vec{B}=-\frac{1}{j\omega }\frac{\partial E}{\partial x}{{\vec{u}}_{z}}$$\Rightarrow i(x,t)=-\frac{a{{c}^{2}}}{j\omega }\frac{\partial \sigma }{\partial x}$
L'équation de Maxwell-Ampère donne: $\vec{E}=-\frac{{{c}^{2}}}{j\omega }\frac{\partial B}{\partial x}{{\vec{u}}_{y}}$$\Rightarrow \sigma =-\frac{1}{j\omega a}\frac{\partial i}{\partial x}$

II.3) Coefficient d'autoinduction:

Pour la longeur dx de ligne, de section droite S=ab, $d{{\varepsilon }_{B}}=abdx\frac{{{B}^{2}}}{2{{\mu }_{0}}}=\frac{{{\mu }_{0}}b}{2a}{{i}^{2}}dx\Rightarrow L=\frac{{{\mu }_{0}}b}{a}$

II.4) Coefficient de capacité:

Pour la longueur dx de la ligne: $d{{\varepsilon }_{E}}=abdx\frac{{{\varepsilon }_{0}}{{E}^{2}}}{2}=abdx\frac{{{\sigma }^{2}}}{2{{\varepsilon }_{0}}}=\frac{b{{q}^{2}}}{2a{{\varepsilon }_{0}}dx}\Rightarrow C=\frac{{{\varepsilon }_{0}}a}{b}$

II.5) Vitesse de phase:

$LC={{\varepsilon }_{0}}{{\mu }_{0}}=\frac{1}{{{c}^{2}}}\Rightarrow {{v}_{\varphi }}=c$; il y a accord avec le précédent résultat.

II.6) Définition de la d.d.p. u(x,t):

$\vec{E}=-\overset{\scriptscriptstyle\rightharpoonup}{\nabla }V-\frac{\partial \vec{A}}{\partial t}$; or le potentiel vecteur est polaire et de ce fait perpendiculaire au plan d'antisymétrie des courants, selon x'x$\Rightarrow \int\limits_{y=b}^{y=0}{\vec{E}.d\vec{l}}=-\int\limits_{y=b}^{y=0}{\vec{\nabla }V.d\vec{l}}=V(x,b,t)-V(x,0,t)=u(x,t)$ E(x,t)=bu(x,t).

II.3.a) Expression de l'impédance caractéristique:

${{Z}_{c}}=\rho \frac{b}{ea}\Rightarrow {{R}_{c}}=\frac{\rho }{e}$ et ${{R}_{c}}=\sqrt{\frac{L}{C}}=\sqrt{\frac{{{\mu }_{0}}}{{{\varepsilon }_{0}}}}\frac{b}{a}\Rightarrow si\ b=a,\ {{R}_{c}}=\sqrt{\frac{{{\mu }_{0}}}{{{\varepsilon }_{0}}}}$
A.N.: ${{R}_{c}}=377\Omega $

II.3b) Calcul numérique de e:

Cuivre: e=45 pm; impossible car e< à la dimension d'un atome.Carbone: e=9,28 µm.

II.8) Puissance moyenne transportée par l'onde:

Vecteur de Poynting: $\vec{R}=\vec{E}\wedge \frac{{\vec{B}}}{{{\mu }_{0}}}=\frac{ui}{ab}{{\vec{u}}_{x}}$ et puissance instantanée: p=Re(u)Re(i)
P=<p>=$\frac{1}{4}(u{{i}^{*}}+{{u}^{*}}i)$=$\frac{1}{2}{{Z}_{c}}\left| {{i}^{2}} \right|\Rightarrow P=\frac{1}{2}{{Z}_{c}}{{I}^{2}}$
L'énergie est dissipée dans l'impédance caractéristique (résistance) par effet Joule.

Partie III-Reflexion d'une onde électromagnétique sur une plaque conductrice:

III.1) Coefficient de reflexion:

On est ramené à la question I.4.a; l'onde progressive peut se propager en aval de la plaque$\Rightarrow r=\frac{1}{3}$

III.2) Expression de l'épaisseur de l'effet de peau:

On suppose la conductivité du métal réelle; c'est possible si la fréquence est nettement inférieure aux fréquences optiques; dans l'équation de Maxwell-Ampère, on peut négliger le courant de déplacement devant le courant ohmique si, en utilisant la notation complexe, $\left\| j\omega {{\varepsilon }_{0}}\vec{E} \right\|<<\left\| \sigma \overset{\scriptscriptstyle\rightharpoonup}{E} \right\|\Rightarrow \rho \omega {{\varepsilon }_{0}}<<1$
Les équations de Maxwell s'écrivent alors:
$ \vec{\nabla }\wedge \vec{B}={{\mu }_{0}}\sigma \vec{E}\Rightarrow -j\vec{k}\wedge \vec{E}={{\mu }_{0}}\sigma \vec{E} $
$ \vec{\nabla }\wedge \vec{E}=-j\omega \overset{\scriptscriptstyle\rightharpoonup}{B}\Rightarrow -j\vec{k}\wedge \vec{E}=-j\omega \vec{B} $
$ \vec{\nabla }.\vec{E}=0(m\acute{e}tal\ neutre)\ et\ \vec{\nabla }.\overset{\scriptscriptstyle\rightharpoonup}{B}=0\Rightarrow \vec{k}.\vec{E}=0\ et\ \vec{k}.\vec{B}=0 $
En reportant le champ magnétique tiré de la seconde équation dans la première et en developpant le produit vectoriel, on obtient: ${{k}^{2}}=-j{{\mu }_{0}}\sigma \omega \Rightarrow k=\pm \frac{1-j}{\delta }\ avec\ \delta =\sqrt{\frac{2}{{{\mu }_{0}}\sigma \omega }}$
On a deux solutions; la solution générale en est une combinaison linéaire.
${{\vec{E}}_{p}}=\left[ {{A}_{1}}{{e}^{-\frac{x}{\delta }}}{{e}^{j(\omega t-\frac{x}{\delta })}}+{{A}_{2}}{{e}^{\frac{x}{\delta }}}{{e}^{j(\omega t+\frac{x}{\delta })}} \right]{{\vec{u}}_{y}}$ et ${{\vec{B}}_{p}}=\frac{1-j}{\delta \omega }\left[ {{A}_{1}}{{e}^{-\frac{x}{\delta }}}{{e}^{j(\omega t-\frac{x}{\delta })}}-{{A}_{2}}{{e}^{\frac{x}{\delta }}}{{e}^{j(\omega t+\frac{x}{\delta })}} \right]{{\vec{u}}_{z}}$

III.3) Relations entre les coefficients:

${{\vec{E}}_{r}}=\alpha {{E}_{0}}{{e}^{j\omega (t+\frac{x}{c})}}{{\vec{u}}_{y}}\ et\ {{\vec{B}}_{r}}=-\frac{\alpha {{E}_{0}}}{c}{{e}^{j\omega (t+\frac{x}{c})}}{{\vec{u}}_{z}}$ et ${{\vec{E}}_{t}}=\tau {{E}_{0}}{{e}^{j\omega (t-\frac{x}{c})}}{{\vec{u}}_{y}}\ et\ {{\vec{B}}_{t}}=\frac{\tau {{E}_{0}}}{c}{{e}^{j\omega (t-\frac{x}{c})}}{{\vec{u}}_{z}}$
Equations de passage: en x=0, il y a continuité du champ électrique tangentiel et du champ magnétique tangentiel(car les courants sont volumiques et non surfaciques);
$ {{{\vec{E}}}_{i}}+{{{\vec{E}}}_{r}}={{{\vec{E}}}_{p}}\Rightarrow (1+\alpha ){{E}_{0}}={{A}_{1}}+{{A}_{2}} $
$ {{{\vec{B}}}_{i}}+{{{\vec{B}}}_{r}}={{{\vec{B}}}_{p}}\Rightarrow (1-\alpha ){{E}_{0}}=\frac{c(1-j)}{\delta \omega }({{A}_{1}}-{{A}_{2}}) $
Equations de passage en x=e:
${{A}_{1}}{{e}^{-(1+j)\frac{e}{\delta }}}+{{A}_{2}}{{e}^{(1+j)\frac{e}{\delta }}}=\tau {{E}_{0}}{{e}^{-j\omega \frac{e}{c}}}$ et $\frac{1-j}{\delta \omega }({{A}_{1}}{{e}^{-(1+j)\frac{e}{\delta }}}-{{A}_{2}}{{e}^{(1+j)\frac{e}{\delta }}})=\frac{\tau {{E}_{0}}}{c}{{e}^{-j\omega \frac{e}{c}}}$

III.4) Expression de :

On considère le seul cas possible: celui du carbone avec e=9,28 µm; $\Rightarrow \delta =\frac{29.8}{\sqrt{\nu }}m$
e<<δ
$\Rightarrow \nu <<{{1,03.10}^{13}}Hz$
; or la formule établie pour δ n'est valable qu'aux fréquences très inférieures aux fréquences optiques; la condition qu'on vient d'obtenir étant déjà nécessaire, l'inégalité e<<δ est donc vérifiée.
Les deux dernières relations de III.3, en utilisant ${{e}^{\varepsilon }}\approx 1+\varepsilon $, conduisent à:
${{A}_{1}}+{{A}_{2}}+\frac{e}{\delta }(1+j)({{A}_{2}}-{{A}_{1}})=\tau {{E}_{0}}{{e}^{-j\omega \frac{e}{c}}}$ et $\frac{c(1-j)}{\delta \omega }\left[ ({{A}_{1}}-{{A}_{2}})-(1+j)\frac{e}{\delta }({{A}_{1}}+{{A}_{2}}) \right]=\tau {{E}_{0}}{{e}^{-j\omega \frac{e}{c}}}$
d'où $\alpha =\frac{-1+j\frac{1}{2}{{\left( \frac{\delta \omega }{c} \right)}^{2}}}{1+\frac{{{\delta }^{2}}\omega }{ce}+j\frac{1}{2}{{\left( \frac{\delta \omega }{c} \right)}^{2}}}$; or $\frac{1}{2}{{\left( \frac{\delta \omega }{c} \right)}^{2}}=\rho {{\varepsilon }_{0}}\omega <<1\ et\ \frac{{{\delta }^{2}}\omega }{ce}=2\frac{{{R}_{c}}}{{{\mu }_{0}}c}=2$$\Rightarrow \alpha \approx \frac{-1}{1+\frac{{{\delta }^{2}}\omega }{ce}}=-\frac{1}{3}$
Mais $r=\frac{1}{3}$ est relatif au coefficient de réflexion de i(x,t), donc de B(x,t) opposé à celui de E(x,t).

III.5) Positionnement d'une plaque de métal parfait:

D'aprèsI.5.b, pour annuler l'onde réfléchie, il faut placer, à la distance $x=e+\frac{\lambda }{4}$, une plaque de métal parfaitement conducteur. On peut le vérifier: sur le métal parfait, on a un noeud de champ électrique, donc en x= e on a un noeud de champ magnétique(propriété des ondes stationnaires) et de ce fait en x=e: $\frac{1-j}{\delta \omega }\left[ {{A}_{1}}-{{A}_{2}}-(1+j)\frac{e}{\delta }({{A}_{1}}+{{A}_{2}}) \right]=0$$\Rightarrow \frac{{{A}_{1}}-{{A}_{2}}}{{{A}_{1}}+{{A}_{2}}}=(1+j)\frac{e}{\delta }$; par ailleurs les deux premières relations de III.3 restent valables$\Rightarrow \frac{{{A}_{1}}-{{A}_{2}}}{{{A}_{1}}+{{A}_{2}}}=\frac{\delta \omega }{c(1-j)}\frac{1-\alpha }{1+\alpha }$; on en déduit $\alpha =0$ comme prévu.

Autres Concours

2011  : Concours ENAC de  physique 2011  :  énoncé ,  corrigé Concours ICNA de  physique 2011  :  énoncé ,  corrigé Concours ICNA de ...